October 2008 LSAT
Section 1
Question 15
Zack's Coffeehouse schedules free poetry readings almost every Wednesday. Zack's offers half–priced coffee all day on...
Replies
Naz on February 3, 2015
This is a Must Be True Question. Thus, the correct answer should be 100% supported by the passage. (D) is better than (E) because answer choice (D) is 100% supported by the passage, whereas (E) is not.Let's diagram!
"Zack's Coffeehouse schedules free poetry readings almost every Wednesday."
So "almost every" is a majority, which we know is "most." So the statement above is saying, on most Wednesdays, Zack's Coffeehouse schedules free poetry readings. We can diagram like so:
Q1: W-most-PR
PR-some-W
"Zack'a offers half-priced coffee all day on every day that a poetry reading is scheduled."
So, if a poetry reading is scheduled, Zack's offers half-priced coffee. We can diagram like so:
P1: PR ==> HPC
not HPC ==> not PR
We know that when the right-hand side variable of a quantifier statement is the same as the sufficient condition of a Sufficient & Necessary statement, then we can connect them. So, we can connect Q1 to P1 like so: W-most-PR ==> HPC to conclude: W-most-HPC, i.e. on most Wednesdays, Zack's will have half-priced coffee.
This deduction is basically what answer choice (D) tells us: "Zack's offers half-priced coffee all day on most if not all Wednesdays."
Answer choice (E) states: "On some Wednesdays Zack's does not offer half-priced coffee all day."
Well, the information in the passage never tells us when Zack's does NOT offer half-priced coffee all day. We merely know that Zack's offers half-priced coffee all day on every day that poetry reading is scheduled. This does not mean that if there isn't a poetry reading scheduled that Zack's will most definitely not offer half-priced coffee all day. We do not have adequate information for this.
Just because we can infer that on most Wednesdays Zack's offers half-priced coffee all day, we cannot infer that on some Wednesdays Zack's does not offer half-priced coffee all day. Remember, a quantifier statement does not have a contrapositive, we can merely reverse the variables and make a "most" into a "some," or vice versa.
So, we only know: W-most-HPC, or HPC-some-W
And remember, "most" means "more than half." So, technically--in our LSAT world--the statement "on most Wednesdays, Zach offers half-priced coffee," could technically mean "on all Wednesdays, Zach offers half-priced coffee." This isn't a Could Be True Question. We are not looking for an answer that could be true. We are looking for the answer choice that must be true, i.e. the answer choice that is 100% supported by the passage.
Thus, since (D) must be true, whereas (E) could be true, (D) is the correct answer.
Hope that clears things up! Please let us know if you have any other questions.
Matt on February 4, 2015
What lesson plan would help me improve on these types of questions?Naz on February 5, 2015
You should make sure to review the lesson on Must Be True Questions and Quantifier Questions.@chris_va on September 23, 2019
I have to comment on this one. In one of the instructions on quantifiers it is stated that some can mean 1. In this question, 1 seems to be closer to the phrase used than most if not all, that fact that it says "all" even though it's broad still seems to exclude it, more so than the possibility of one -less than 50%. Thanks LSATMax.kbernard on June 25, 2020
So can we always assume that almost everything means "most" and not "some"? I also remember the explanation for another question where almost all meant someheaven11953@gmail.com on August 15, 2020
what is the difference between answer choice a and b?Victoria on August 19, 2020
Hi @kbernard,Always make sure to read the words within the context of the stimulus. However, it is usually a safe bet to assume that "almost everything" means "most" and not "some."
Remember that "most" means more than half whereas "some" covers more than one but less than half. Therefore, "almost everything" would be covered by "most" as it suggests the vast majority.
Hope this is helpful! Please let us know if you have any further questions.
Victoria on August 19, 2020
Hi there,Answer choices (A) and (B) are different because they focus on two different aspects of the stimulus.
Answer choice (A) focuses on half-priced coffee whereas answer choice (B) is focused on free poetry readings. While we know that Zack's offers half-priced coffee every day that a poetry reading is scheduled, the key difference between these answer choices is the difference in their focus.
Answer choice (A) is incorrect because we are not given any information regarding the other days on which Zack's may or may not offer half-priced coffee. It is entirely possible that Zack's offers half-priced coffee every Saturday, making it the most common day as free poetry readings only occur almost every Wednesday.
Answer choice (B) is incorrect because the stimulus does not tell us whether Wednesdays are the only day on which free poetry readings are scheduled. As with the above, it is entirely possible that Zack's schedules free poetry readings every Saturday, meaning that most poetry readings would be scheduled on Saturday and not Wednesday.
Hope this helps! Please let us know if you have any further questions.